Sei sulla pagina 1di 33

Law Of Contracts-I

Paper-1 Contract I 3- Hours 100


Law of Contracts
Q. 1 Explain the difference between Proposal and Invitation for proposals. What is a
general offer? Define Contract. What are the essential elements of a contract?
Section 2(a) says that when a person signifies to another, his willingness to do or to abstain from doing
something with a view to obtaining the assent of that another to such act or abstinence, he is said to make a
proposal. Thus, when an offer is made with such finality that the only thing remaining is the assent of the party
to whom it is proposed, it is called a proposal. If the offer does not reflect such finality, it is merely an invitation
to proposals. Thus, when a person advertises that he has books to sell, it is only an invitation for proposals.

When a person, without expressing his final willingness, proposes certain terms on which he wishes to
negotiate, he is merely making an invitation to proposals.
In the case of Harvy vs Facie 1893, plaintiff telegraphed, "Will you sell Bumper Hall pen? Send lowest price."
Defendants responded with "Lowest price of Bumper Hall Pen, $900". Plaintiffs then sent, "we agree to buy
bumper hall pen for $900". However, defendants refused to sell. It was held that defendants had not signified
a final willingness to sell. They had only told the lowest price. This, it was only an invitation to proposal and not
a proposal.

In the case of Pharmaceutical Society of GB vs Boots Cash Chemists Ltd. 1952, it was held that display
of goods is also an invitation to sell even if it is a self-service shop.

MC Pherson vs Appanna 1951 - Proposal to buy property at 6000/- was replied with, " won't accept less than
10000". This was not considered a proposal but an invitation to proposal.

Auctions : Announcement to hold auction is not an offer. Highest bid is nothing more than an offer to buy.

Definiteness of proposal: "Cocks and Hens - 25s each" is not an offer to sell.

Q. Define contract. State essential elements of a valid contract.


Section 2(h) of Indian Contract Act 1972 - An agreement enforceable by law is a contract.
Def. by Anson - A contract consists in an actionable promise or promises.
Def. by Sevigni - Contract is a combination of agreement and responsibility.
Def. by Pollock - An agreement or promise enforceable by law is contract.

Essential Elements
Section 10 - All agreements that are made by people competent to contract, with free consent, for a lawful
object and lawful consideration and not expressly declared to be void are contracts.

Thus, we get the following elements -


1. There must be two or more people involved.
2. There must be an intention to contract. Balfour vs Balfour 1919 - Husband promised to send money
to wife. Not a contract because there was no intention to contract.
3. There must be an agreement to do or to abstain from doing something.
4. The agreement must involve a lawful purpose, which means - agreement must not be against
marriage, trade, legal proceedings, or it must not be a wagering agreement or must not be expressly
prohibited by law.
5. Agreement must not be uncertain
6. Must not be impossible. Sec 56.
7. Free consent - not vitiated by coercion, undue influence, fraud, or misrepresentation.
8. Parties must be competent to contract.
9. Lawful consideration.
10. Lawful object.
11. Legal formalities
12. Must be enforceable by law, which means must not be immoral or against public policy.
Law Of Contracts-I

Q. What is a general offer? How is a contract created through


general offer? Refer to leading cases.
An offer may be made to the world at large. Such an offer is a general offer. However, a contract is not done
with the whole world but only with the person who comes forward and accepts the offer. The acceptance might
be express or implied.
As per Anson, "An offer need not be made to an asertained person, but no contract can arise until it is
accepted by an ascertained person".

Case of Carllil vs Cabolic Smoke Ball Company.

Creation of the contract - If the person performs the conditions of the offer. Thus, a person who finds a lost
dog fulfills the condition of the prize money and thus a contract with the owner of the dog is created.

General Offer of Continuing Nature - Some offers such as finding a lost object close when it is accepted by the
first person. However, some offers, such as in the Carllil case, it can be accepted by any number of persons
until the closing date of offer or until it is retracted.

Q. 2 Describe the law relating to communication, acceptance and their revocation.

Section 2(a) of Indian Contract Act 1972 says that when a person signifies his willingness to do or to abstain
from doing something to another, with a view to obtaining the assent of that another, he is said to make a
proposal. Further, section 2(b) says that when the person to whom the proposal is made signifies his assent,
the proposal is said to be accepted. The important point to note here is that the party making the proposal or
the party accepting the proposal must "signify" their willingness or assent to the other party. Thus, a promise
cannot come into existence unless the willingness or assent is communicated to the other party. Further, even
the revocation, if any, must be communicated to the other party for it to take effect. Therefore, communication
is the most critical aspect in the making of a contract.

Communication
Section 3 defines how a communication, acceptance, or revocation can be signified:
The communication, acceptance, and revocation are deemed to be made by an act or omission of the party
proposing, accepting, or revoking, by which he intends to communicate such proposal, acceptance, or
revocation, or which has the effect of communicating it.
Thus, a proposal may be made by any way, which has the effect of laying before another person his
willingness to do nor not do something. The acceptance can be signified similarly. Section 9 specifies that a
promise (i.e. a proposal and its acceptance) can be formed either by words, written or oral, is which case it is
called express or by action, in which case it is called implied. In the case of Haji Mohd Ishaq vs Mohd Iqbal
SCC 1978, the defendants accepted the goods supplied by the plaintiff through a go between man and also
paid part of the price. It was held that the defendants were liable to pay the remaining balance because the
proposal and its acceptance were signified by their actions.

Section 4 specifies when a communication is complete:

Communication of a proposal is complete when it comes to the knowledge of the party to whom the
proposal is made.

For example, if A sends a proposal in the mail to B and if the mail is lost, it can be held that the communication
of the proposal is not complete. In the case of Lalman vs Gauridatta 1913, it was held that the reward for the
missing child cannot be claimed by a person who traced the child without any knowledge of the
announcement. There was no contract between the two in the first place because the proposal never came to
the knowledge of the person who found the child and thus he could never accept it.
Communication of the acceptance is complete, as against the promisor, when it is put in course of
transmission to the promisor so as to be out of the power of the acceptor, as against the acceptor,
when it comes to the knowledge of the promisor.
Law Of Contracts-I

For example, as soon as B drops a letter of acceptance in mail back to A, A is bound by the promise.
However, B is not bound by it unless A receives the acceptance letter. In the case of Adams vs Lindsell
1818, it was held that a contract arose as soon as the acceptance was posted by the acceptor. In this case,
the plaintiff received the offer to sell wool on 5th and they posted an acceptance, which was received on 9th
by the defendants. The defendants, however, had already sold the wool on 8th. The court observed that the
contract must arise as soon as the acceptance is posted and is gone out of the reach of acceptor otherwise
this will result in an infinite loop.
Communication of a revocation is complete as against the party who makes it when it is put in course
of transmission to the party to whom it is made, so as to be out of the power of the party who makes
it; as against the party to whom it is made, when it comes to the knowledge of the party to whom it is
made.

For example, if A sends a letter revoking his proposal, it will be complete against A as soon as the letter is
dropped in the mailbox and is out of his control. However, the revocation will be held complete against B only
when B receives the letter.
Further, if B revokes his acceptance by telegram, it will he deemed complete against B as soon as he
dispatches the telegram. It will be held complete against A, when A receives the telegram.

Section 5 specifies when a proposal and acceptance can be revoked:


A proposal can be revoked anytime before the communication of its acceptance is complete as
against the proposer but not afterwards.

For example, if A propose to B through a letter, A can revoke the proposal as long as B has not posted a letter
of acceptance to A. In the case of Henthorn vs Fraser 1862, an offer to sell a property was made to a person.
This person was to reply to it within 14 days. He lived in another town and he posted an acceptance at
3.50PM, which reached the offerer at 8.30 PM. Meanwhile, the offerer posted the revocation letter at 1 PM,
which reached the person at 5.30PM. Thus, the revocation did not reach the offeree before the communication
of the acceptance was complete as against the offerer. Thus, the revocation was held ineffective.

An acceptance may be revoked anytime before its communication is complete as against the
acceptor.

For example, B can revoke his acceptance that was sent by letter, by a telegram that reaches A before the
acceptance letter. In the case of Union of India vs Bhimsen Walaiti Ram 1969, the defendant won an
auction for a liquor shop and paid 1/6 of the cost upfront. However, the bid was supposed to be finalized by
the financial commissioner, which he had not done. Meanwhile, the defendant failed to pay the remaining
amount and the commissioner ordered a re-auction. In the re-auction, less money was realized and the
plaintiff sued to recover the shortfall. However, SC held that since the commissioner had not given is final
approval for the bid, the communication of acceptance was not complete against the defendant, thus the
defendant was free to withdraw or revoke his proposal (i.e the bid).

Section 6 specifies how a revocation can be made:


A proposal is revoked
o by the communication of the notice of revocation by the proposer to the other party.
o by the lapse of prescribed time in the proposal for acceptance or if no time is prescribed, by
the lapse of a reasonable time in communication of the acceptance.
o by the failure of the acceptor to perform a condition precedent to acceptance.
o by death or insanity of the proposer, if the fact of the death or insanity comes to the
knowledge of the acceptor before acceptance.

Acceptance

Section 7 specifies that an acceptance must be absolute and unqualified. A partial acceptance or a
clarification regarding a proposal, or specifying a condition on acceptance is no acceptance.
In the case of Hyde vs Wrench 1840, an offer was made to sell a farm for #1000, which was rejected by an
plaintiff, who counter offered #950 for it. This was rejected by the defendant, upon which the plaintiff agreed to
pay #1000. However, it was held than the defendant was not bound by any such second acceptance.

Section 7 further says that the acceptance must be in some usual and reasonable manner, unless the
Law Of Contracts-I

proposal prescribes the manner in which the acceptance should be made. If the proposal prescribes the
manner, and if the acceptance is not done in that manner, the proposer may insist that the acceptance be
made in the manner prescribed, and if he fails to do so, he accepts the acceptance. Thus, if the acceptance is
sent by any way other than what is prescribed by the proposal, the proposer must reject it in a reasonable
time otherwise the proposer accepts it. This is markedly different from English law where a proposal must be
accepted in the manner required in the proposal otherwise, the acceptance is invalid. In the case of Elliason
vs Henshaw 1819, it was held that an acceptance sent by mail instead of through the wagon that brought the
offer, was not valid.

Section 8 specifies that a proposal is accepted when the acceptor performs conditions prescribed for the
acceptance or when he accepts the consideration given along with the offer for a reciprocal promise. When
acceptance consists of an act as in the case ofState of Bihar vs Bengal C & P Works 1954, it was held that,
when an order is sent for goods, the posting of goods itself is equivalent to acceptance. No further
communication of acceptance is necessary.

In the case of Carlill vs Carbolic smoke ball co 1893, it was held that, purchasing and consuming the
medicine performs the condition of the proposal.

Requirements for an acceptance

1. Acceptance must be from a person to whom the proposal was made. In the case of Powel vs Lee
1908, it was held that communication of an acceptance from an unauthorized person is invalid.
2. Acceptance must be signified to the proposer. In the case of Felthouse vs Bindley 1863, it was held
that unless an acceptance is given to the offerer, it is no acceptance.
3. It is required that there be an act that signifies the acceptance. As held in the case of Bhagvandas
Goverdhandas Kedia vs Girdharilal Pursottamdas & Co SC AIR 1966, for an acceptance to be
completed, a mere mental decision is not sufficient. An external manifestation of the decision is a
must.

Communication and acceptance of General Offers


A general offer, such as an advertisement for the sale of an article at a fixed price, or to give prize to the one
that does something first, is not made to a particular person. Whoever the contract is done with the person
who responds or who does the task first. Communication of such as offer is done through public media such
as a newspaper. S general offer can be perpetual or end as soon as the condition is fulfilled.
No explicit acceptance of such offers is usually required. Performing the conditions specified in the offer acts
as the acceptance of the offer. For example, in the case of Carlill vs Carbolic Smoke Ball Company 1893, it
was held that it was a general offer and anybody who fulfilled the condition was eligible for the $100
compensation as advertised.

Revocation of General Offers


A general offer can be revoked in the same manner as it was made. For example, by printing a revocation in a
newspaper. It will be considered complete, even if a person who is ignorant of the revocation, performs the
conditions after the revocation is published.

Q. 3 "An agreement enforceable by law is a contract" as per section 2(h) of Indian Contract
Act 1872 but "agreements enforceable by law have been defined in section 10". Who are
competent to contract? What are the provisions for a contract with minor?

In our regular day to day life we make several comments and statements. We say several things to people
whom we talk to. Most of these are not with any intention to create any legal obligation. For example, if we say
to someone that we will go to lunch with him, it is not a legal obligation. But some, which are related to
business or civil matters, are understood to be in a serious mood and have a potential to be legally
enforceable. For example, when we hire an Auto-rickshaw for going from point A to point B, we are legally
bound to pay and the driver is legally bound to take us from A to B. Indian Contract Act 1872 defines these
activities in precise terms in Section 2.
Law Of Contracts-I

Definition of Terms
Sec. 2 (a) When a person signifies to the other, to do something or to abstain from doing something, with a
view to obtaining the assent of that other to such act or abstinence, he is said to make a Proposal.
Sec. 2 (b) When the person to whom the proposal is made signifies his assent thereto, the proposal is said to
be accepted. An accepted proposal becomes a promise.
Sec. 2 (c) The person making the promise is called Promisor, while the person accepting the promise is called
Promisee.
Sec. 2 (d) When, at the desire of the Promisor, the Pomisee or any other person has done or abstained from
doing, or does or abstains from doing, or promises to do or abstain from doing, something, such act or
abstinence is called a consideration for the promise.
Sec. 2 (e) Every Promise and every set of Promises forming a consideration for each other, is an Agreement.
Sec. 2 (f) Promises which form the consideration or part of consideration for each other are "Reciprocal
Promises".
Sec. 2 (g) An agreement not enforceable by law is void.
Sec. 2 (h) An agreement enforceable by law is a Contract.
Sec. 2 (i) An agreement that is enforceable by law at the option of one or more of the parties thereto but not
at the other or others is a voidable Contract.
Sec. 2 (j) A Contract that ceases to be enforceable by law becomes void when it ceases to be enforceable by
law.

From sec 2(e) and 2(h), it is clear that Agreement and Contract are two different things. For an agreement to
become a contract, it has to be enforceable by law.
Section 10 states that all agreements that are made by free consent of the people who are competent to
contract, for a legal object and legal consideration, and are not hereby expressly declared to by void, are
contracts and are thus legally enforceable. Thus, there are five factors that determine whether an agreement
can be legally enforced or not. These are discussed below:

1. Competency of the people doing the agreement. (What do you


understand by competency to contract? Who are competent to
contract? What protections are offered to minors?)
All the parties doing the agreement must be competent to contract. Section 11 determines who are competent
to contract. As per this section, person who has attained the age of majority according to the law to which is
subject, who is of sound mind, and who is not prohibited/disqualified from contracting by law to which he is
subject. Majority is 18 years except when a guardian is appointed by the court in which case it is 21 yrs.

In the case of Mohoribibee vs Dharmodas Ghosh in 1903, a minor had taken a loan and then he sued to
avoid the contract. Privy Council council held that any contract with a minor is void ab initio and so the loaner
cannot get any money that he gave as advance back. This rule is adopted all over India whether or not it
benefits the minor.
In the case of Mir Sarwarjan vs Fakhruddin Mohd. Chaudhary 1912, a contract to purchase a property was
done on behalf of minor. It was held that the minor could not sue for getting the possession of property.

However, since in today's times minors are coming a lot in public life, it is not always possible to consider an
agreement with a minor to be always void. Therefore, in the case of Srikakulam Sbhramanyam vs Kurra
Sabha Rao 1949, Privy Council held that a sale of inherited property of a minor to pay off inherited debt
effected by the guardian was binding on the minor.

Protections offered to minors:


No estoppel against minor - It has now been settled that a minor who enters into a contract by
misrepresenting his age can later on tell his correct age and avoid the contract.

No liability in tort or in contract arising out of a contract - If a minor enters into a contract, he can neither
be held liable in contract nor in torts. In the case of Jennings vs Rundall 1799, when an infant hired a horse
for riding short distance but rode it for long distance resulting in injury to horse, he was not held liable because
it was a contractual obligation. In the case of Hari Mohan vs Dulu Mia 1934, Calcutta HC held minor not
liable in tort for money lent on bond.
However, in absence of a contract, a minor may be liable in tort. Thus, in the case of Burnard vs Haggis
1863, when a minor "borrowed" a mare only for riding and then lent it to a friend who jumped her and killed
Law Of Contracts-I

her, he was held liable in tort.

Doctrine of restitution - If a minor obtains property or goods by misrepresenting his age, he can be forced to
return it but only as long as the goods are traceable in the minor's possession. This is called doctrine of
equitable restitution. If the minor sells or converts the property, the value of the goods cannot be retrieved
because that would amount to enforcing a void contract. In the case of Leslie vs Sheill, a minor got 400
pounds from money lenders by misrepresenting his age. The money lenders could not recover it under any of
fraud, quasi-contract, or doctrine of restitution. This was followed in the case of Mohoribibee vs Dharmodas
Ghosh as well..

Beneficial Contracts - In contract where a minor has already supplied consideration, the minor can enforce
the contract. Thus, in the case of Ulfat Rai vs Gauri Shakar 1911, it was held that a minor can sue to take
possession of a property for which he has already paid. But where the contract is still executor and
consideration has not been given, the principle adopted in Mohoribibee will prevail. Thus, in the case of Raj
Rani vs Prem Adib 1949, it was held that the film producer was not bound by a contract with minor's father to
give a role to minor in his movie. This is because minor could not be forced to give consideration and father
had not given any consideration. However, a contract of marriage of a minor enter into by the father is not void
for want of consideration because it is for the benefit of the minor.

Liabilities for necessities (Section 68) - If a minor is supplied with necessaries that are in accordance with
his living standard, the supplier can get paid through the minors property.

Persons of unsound mind


Section 12 says that a person is of sound mind for the purpose of contracting if at the time of contracting, he
is capable of understanding the contract and capable of making a rational judgement as to the effects of the
contract upon his interests. A person who is usually of sound mind but sometimes of unsound mind may not
make a contract when he is of unsound mind, while a person who is usually of unsound mind but sometime of
sound mind may make a contract when he is of sound mind. Thus, a person, who is too drunk, or who is
temporarily delirious due to sickness such as high fever, may not make a contract at that time. A patient in a
lunatic asylum, who is at intervals of sound mind may make a contract when he is of sound mind.
In India, a contract done by a person of unsound mind is absolutely void ab initio. In the case of Indersingh
vs Parmeshwardhari Singh Patna HC in 1957 held that a contract to sell property worth 25000 in 7000, was
voidable because the mother claimed that her son was of unsound mind and did not understand the
implications.

2. Consent and Free Consent


Section 13 defines that two or more people are said to consent when they agree upon the same thing in the
same sense. However, many a times, a consent may not reflect the true intentions of a party. For example,
one party may give consent because of being financially pressured or criminally threatened. Thus, such a
consent should not make the agreement enforceable. Section 14 determines what factors can vitiate a
consent and when a consent is considered free of any complication that affects the enforceability of an
agreement . It states that a consent that is not obtained through coercion, undue influence, fraud,
misrepresentation, or mistake subject to section 20, 21, and 22, is a free consent.

a. Coercion (Sec 15): Coercion is committing or threatening to commit any act forbidden by the Indian Penal
Code, or unlawful detaining or threatening to detain the property, to the prejudice of any other person, with an
intention to cause that other person to enter into an agreement. It is immaterial whether IPC is or is not in
force where coercion is applied. Thus, an act that is unlawful as per IPC but not as per England law and that
has been used to induce the consent, will be considered coercion.
A clear example would be force someone to consent on gun point or by hurting or threatening to hurt.
In Chikham Amiraju vs Chikham Seshamma Madras HC 1912 held that threatening to commit suicide is
coercion. In the case of Astley vs Reynolds 1771, the plaintiff had pledged his plate for #20 and when he
went to claim it back, the defendant asked for #10 more as interest. To redeem his plate, the plaintiff paid the
money but later sued to recover #10. The court allowed it.

b. Undue Influence (Sec 16): Undue influence occurs when because of the nature of the relationship that
exists between the parties, one party is able to dominate the will of the other and uses this dominance to
obtain unfair advantage over the other. A person is in a dominant position when he holds a real or apparent
position of authority for example manager employee, or stands in a fiduciary relationship with the other for
example money lender and loanee. A person could also be in a dominant position if the mental capacity of
other party is temporarily or permanently effected due or illness, age, or distress.
The burden of proof that undue influence has not occurred is on the person who is in the dominant position, if
Law Of Contracts-I

the agreement is unconscionable otherwise it is on the party that alleges undue influence.
Examples:
Father (A) give some money to son (B) when B was a minor. Upon majority, A makes B execute a bond for
a much larger amount.
A person (A) who is old and sick is induced into paying an unreasonably large amount of sum to his doctor
(B).
A village moneylender (A) lends money to a villager (B), who is already in debt, at a very high interest. It lies
on A to prove that he has not used undue influence to induce the contract.
At a time of financial crises, a bank manager gives loan to a person at a substantially higher rate. This is not
considered to be undue influence but a simple business transaction.
In Mannu singh vs Umadat Pandey Allahbad HC 1890, a guru induced his devotee into giving all the
devotee's property to himself. This was considered undue influence.

c. Fraud (Sec 17): When a person intentionally tries to cheat another person, it is called as fraud in a general
sense. Section 17 defines fraud precisely as such - Fraud means and includes any of the following activities
done by a party or by his connivance or by his agent, with an intent to deceive another party or his agent, or
as to induce the other party to enter into the contract.
1. the suggestion of a fact, of that which is not true, by the one who does not believe it to be true.
2. active concealment of a fact by one who knowledge or belief of the fact.
3. making a promise without an intention to perform.
4. any act fitted to deceive
5. any such act or omission that the law declares to be fraudulent.

Mere silence as to facts likely to affect the willingness of a person to enter into the contract is not fraud unless,
according to the circumstances of the case, it is the duty of the person keeping silence to speak or unless his
silence itself is considered as speech.
Examples:
A sells a horse to B by auction without telling B that horse is unsound. This is not fraud.
B is A's daughter who has just come off age, then it is A's duty to tell B about the fact. So this is fraud.
B says to A, "if you do not deny it, I will assume that horse is sound". Here, silence is considered as speech so
this is fraud.
A and B, being traders, enter into a contract. A has private pricing information that will cause B to not enter the
contract. A is not bound to inform this to B. This is not fraud.
Concealing the disease history while obtaining insurance is fraud because it is the duty of the insured to give
this information to the insurer.
Derry vs Peek 1889 was not fraud, because the company honestly believed in what they said and there was
no intentional misrepresentation, which is the essence of fraud.
Sri Krishan vs. Kurukshetra Univ., AIR 1976 SC the student was not found to be fraud. Even though he
knew that he was short on attendance, he did not disclose it on examination form. He was let off because
'mere silence' is not fraud.

d. Misrepresentation (Sec 18): When a person makes an unwarranted statement, however innocently, which
the person believes to be true, and which turns out to be false, it is misrepresentation. Any breach of duty,
without an intention to deceive, that gains an advantage to the person committing it or to the person claiming
under him, by misleading the other person to his prejudice or to person claiming under him, is also
misrepresentation. Further, causing a party to an agreement to make a mistake regarding the subject matter
of the agreement, however innocently, is also misrepresentation.
Examples:
A claimed to B that the ship being considered under an agreement was below 2800 tonnage. But in reality it
turned out to be more than 3000 tonnage. It was held to be misrepresentation and B was entitled to avoid the
contract. Oceanic Steam Navigation vs Soonderdas Dharmasey. Bom HC 1980.
A land was purchased expressly for constructing duplexes. The seller claimed that he saw no permissioning
problems. However, later on the permission was denied. This was held to be misrepresentation and even
though the claim was innocent, the buyer was allowed to avoid the sale
Where the seller of a car stated the mileage of the car to be 20000, which turned out to be wrong, the buyer
of the car was allowed to recover compensation for misrepresentation.

Section 19 declares that a contract induced due to coercion, fraud, or misrepresentation is voidable at the
option or the party whose consent was obtained by coercion. An exception is that when the consent is
obtained by silence fraudulent under sec 17, and when the affected party had the means of discovering the
truth with ordinary diligence. In this case, the contract is not voidable. Further, if the fraud or
misrepresentation did not cause the party on which they were practiced to give consent, then the contract will
Law Of Contracts-I

not be voidable.
Section 19A declares that the party whose consent was obtained by undue influence has the option to avoid
the contract.

3. and 4. Legal Object and Legal Consideration


When four dacoits enter into an agreement to share the loot equally and if two of them take more share than
the other two, there is nothing that law can do, except arrest them for dacoity. Needless to say, the objects
and considerations involved in the the agreement have to be legally valid. Section 24 declares that
agreements are void if consideration and object are unlawful in part. If any part of a single consideration for
one or more objects, or any one or any part of one of several considerations for a single object, is unlawful,
the agreement is void.
Thus, if the unlawful part cannot be severed from the object or consideration, the whole agreement becomes
void. However, if the unlawful part can be severed, the remaining part can still be enforced. For example, A
enters into an agreement with B to get 1 Gram of Cocain with 1 Kg or Rice for 10 Rs for rice and 1000 Rs for
Cocain, then a part of the object that is 1 gm of cocain and 1000Rs is severable from the agreement without
affecting the lawful part. In this case, the agreement can be enforced partially. In another example, A man
enters into an agreement with a married women to clean his house and live with him in adultery, which is
unlawful, for a sum per month, the whole agreement is void because it is not possible to divide the sum
properly between lawful and unlawful objects.

What objects or considerations are unlawful


Section 23 declares that any object or consideration is lawful, unless - it is forbidden by law, or is of the nature
that if permitted, defeats the provisions of any law , or is fraudulent, or implies or involves injury to the person
or property of another, or is determined by the court to be immoral or against public policy. Thus, an
agreement to rent an apartment for prostitution or gambling is void.

5. Agreements expressly declared to be void by this act


Some other agreements that satisfy all the four conditions given is section 10 can still be void. Such
agreements are:
Sec 20 : when both the parties are under mistake as to the matter of fact, the agreement is void.
Sec 24 : agreement in which any part of a single consideration for one or more objects, or any
consideration or part of a consideration out one or more considerations for a single object is unlawful, is void.
Sec 25 : agreement without any consideration except if it is registered, or a promise to pay for something
already done, or is a promise to pay time barred debt.
Sec 26: agreement in restraint of marriage.
Sec 27 : agreement in restraint of trade.
Sec 28 : agreement against legal proceedings.
Sec 29 : agreement that is uncertain
Sec 30 : agreement by way of wager.
Sec 56 : agreement to do impossible act is void. If an act becomes impossible after the contract it done, the
contract becomes void when the act becomes impossible to do.
Legal formalities: Certain agreements such as agreement for the sale of immovable property, or agreement for
insurance become a contract only when they are properly registered. For such agreements, the procedure
prescribed by law must be followed to make them a contract.

Q. 4 Explain - Coercion, Undue Influence, Fraud, Misrepresentation?

According to Section 10, free consent is an integral part of a contract. An agreement cannot become a
contract unless it is done by free consent of parties.
Section 14 says that a consent is free when it is not vitiated by coercion, undue influence, fraud,
misrepresentation, or by mistake subject section 20, 21, 22.

Coercion
Section 15 defines coercion as follows -
Coercion is committing or threatening to commit an act that is prohibited by IPC, or any unlawful detaining or
threatening to detain, any property, to the prejudice of any person whatever, with an intention of causing any
Law Of Contracts-I

person into entering a contract. It is immaterial whether IPC is in operation at a place where such act took
place.

Illustrations
A threatens B at gun point to sell his land to A.
A while in an English ship on high seas enter into a contract with B by intimidating B that is unlawful in India.
Later on A sues B of breach of contract in Calcutta. This is coercion.

Chikham Amiraju vs Chikham Seshamma 1912 - Husband threatened to suicide unless wife gave property
to his brother. This was held coercion.
Askari Mirza vs Bibi Jai Kishori 1912 - Threatening a criminal prosecution is not coercion per se. It could
be coercion if the threat is to file false charges.
Astley vs Reynolds 1731 - Plaintiff had pledged his place for $10. When he went to take it back, pledgee
asked for $10 more. He paid the additional $10, but sued to get recover it back. It was held coercion.
Andhra Sugars vs State of AP 1968 - A factory was bound to take the sugar cane from the farmer under an
act. This was not held to be coercion.

Undue Influence
Section 16 defines Undue Influence as follows -
A contract is said to be induced by Undue Influence when the relationship between the parties is such that one
party is able to dominate his will on the others and uses that position to gain an unfair advantage. A person is
deemed to be in the position of dominating the will of the other if -
if the person holds a real or apparent position of power
If stands in a fiduciary relationship with the other.
if the other person is mentally weak because of sickness, disease, or economic distress

It further says that if a contract is unconscionable the burden of proof lies of the person in whose favor the
contract is to prove that it was not induced by Undue Influence, other wise the burden of proof is on the one
who alleges it.

Illustrations
A advances some money to his minor son B. Upon majority, A makes B sign a contract to pay back more than
the sum advanced.
A is sick and physically feeble and is attended by his nurse B. B influences A to enter a contract to pay him an
unreasonable amount for his professional services.
A being in debt of B, the village money lender goes to B for getting a loan. B gives the loan on terms that are
unconscionable. It lies on B to prove that undue influence was not used to create the contract.
A applies for loan to a banker B while there is a stringent crises in the money market. B declines to give the
loan only at a very high rate. This is not coercion but simple business transaction.

Ability to dominate the will


Mannu Singh vs Umadat Pandey 1890 - Spiritual guru induced the plaintiff, his devotee, to gift all his
property to the guru.

Relations of dominion over other - parties are not on equal footing.


Williams vs Baylex 1866 - father being afraid of bank manager, entered into a contract to mortgage his
house. This was held voidable.

Real of Aparant authority


Income tax office, magistrate, police officer etc.
It may not be real but if a person shows off as such then also it is applicable.

Fiduciary Relation
Every relationship of trust and confidence is a fiduciary relationship.
Solicitor - client, doctor - patient, spiritual guru - devotee.

Mental Distress
Ranee Annapurni vs Swaminatha 1910 - A poor widow who was in dire need to money to establish her right
to maintenance, was persuaded by a money lender to take loan at the rate of 100%. It was held to be undue
influence while a person was under mental distress and the court reduced the rate to 24%.
Law Of Contracts-I

Burden of Proof - The person must show that the other party was in position of dominating the will and that
he used that position to gain advantage.

Presumption of undue influence


In certain cases, when it is established that the defendant was in a position to dominate the will of the plaintiff,
it will be presumed that he must have used his position to obtain an unfair advantage. Thus, it will be up to the
defendant to prove that the plaintiff freely consented.

Lancashire Loans Ltd. vs Black 1934 : It was held that a daughter may not necessarily be independent and
may be under the influence of the mother.

Presumption is raised in the following cases -

1. Unconscionable bargains
Wajid Khan vs Raja Ewaz Ali Khan 1891 - An old illiterate woman conferred upon her managing agent a bug
pecuniary benefit without any valuable consideration under the guise of a trust. This was held to be under
undue influence.

2. Inequality in bargaining power


LLoyd's Bank vs Bundy - Farmer pledged his farmhouse for securing a loan for his son. Later bank tried to
take possession of the house. It was held that the contract might have been done under undue influence.

3. Contracts with Pardanashin women


A contract with a pardanashin woman is presumed to have been induced by undue influence. However, such
a woman must be totally secluded from ordinary society. In the case of Ismail vs Amir Bibi 1902, a lady stood
as witness, put tenants, collected rents in respect of her house. She was held not a pardanashin woman.

Coercion - Section 15 Undue Influence - Section 16


There is no outward sign on undue
There is a clear threat involved and the person being coerced
influence and the person being influenced
knows it.
may not realize it.
A relationship that allows a person to
No relationship has to exist.
dominate the will of other must exist.
Contract induced by undue influence is
Contract induced by coercion is voidable under section 19
voidable under section 19-A
If an act, which is unlawful under IPC, has been used to force a
person into contract, it is immaterial where it has taken place,
contract will still be voidable in India.

Fraud
Section 17 defines fraud as follows:
Fraud means and includes any of the following acts done by a party to a contract, or by his connivance, or by
his agent, to decieve another party thereto or his agent, or to induce him to enter the contract. Such acts
include-
1. the suggestion, as a fact, that of which the party knows or has reason to believe to be not true.
2. active concealment of a fact by the one who knows or has reason to believe to be true.
3. Making a promise he does not intend to fulfill.
4. any act fitted to deceive.
5. any act or omission as the law specifically declares to be fraudulent.

Illustrations
A sells, by auction, to B a horse which A knows to be unsound but does not tell anything to B. This is not fraud.
B is A's daughter who has just come of age. In this case, it is A's duty to tell B that the horse is unsound.
B says to A, "If you do not deny it, I will assume that the horse is good.". Here, A's silence is equivalent to
speech.
A and B are both traders and A has private information about change in prices, which would affect B's
Law Of Contracts-I

willingness to proceed with contract. This is not fraud.


Intention to deceive is required to constitute fraud.

Suggestion of a fact
Derry vs Peek 1889, it was held not to be fraud because the defendants truly believe that permission would
be granted by the board of trade because parliament had approved it.

Active concealment
Active concealment is different from passive concealment. Passive concealment merely means silence as to
material facts. However, active concealment means making efforts to prevent the facts from reaching a party
and this is fraud.
B R Chaudhary vs IOC 2004 - A dealer concealed his previous employment under govt. to get dealership. SC
allowed the contract to be terminated.
Concealment by mere silence is not fraud
Sri Krishan vs Univ. of Kurukshetra 1976 - the candidate knew that he was short of attendance but did not
write anything on the examination form. It was held not fraud because it was the job of the university to
scrutinize the forms.

Silence may become fraud in certain cases - Duty to speak, Half truth, change of circumstances.

Making a promise without any intention to perform


DDA vs Skipper Construction Company 2000- A builder collected deposit money from more number of
people than there were flats. SC held that since the builder knew that he cannot perform his promise and still
took the money, he was doing fraud. He was held liable to pay interest even though there was no provision of
interest on deposit.

Any other act fitted to deceive


Ningawwa vs Byrappa 1968 - Husband got his illiterate wife to sign papers saying that he was mortgaging
her two lands but actually he mortgaged four. This act was obviously done to deceive and was held to be
fraud.

Misrepresentation
Section 18 defines misrepresentation as follows:
Misrepresentation means and includes
1. making a statement in a manner that is unwarranted by the information of the person making it, of
that which is not true, though he believes it to be true.
2. any breach of duty which, without an intention to deceive, gains an advantage to the person
committing it or any one claiming under him, by misleading another to his prejudice or to the
prejudice of anyone claiming under him.
3. causing, however innocently, another party to commit a mistake as to the substance of the thing
which is the subject of the agreement.

Thus, when there is no intention to deceive but still a wrong statement has been made, or a duty has not been
performed, or a mistake has been induced, it is misrepresentation.

Unwarranted Statements
Oceanic Steam Navigation vs Soonderdas Dharmasey 1980 - the defendants charted a ship from a
company. The plaintiff had made a claim that the ship was not more than 2800 tonnage even though the
plaintiff had not known about it. In reality the ship turned out to be more than 3000 tonnes. It was held to be
misrepresentation and the defendants were allowed to avoid the contract.

Breach of Duty
Thake vs Maurice 1986 - Husband was not informed of the risks and failure rate of vasectomy before the
operation. Later on wife became pregnant and the hospital was held guilty of misrepresentation and was
ordered to pay compensation for all the pains and expenses of delivery.

Inducing mistake about subject matter


The subject matter of the contract is supposed by the parties to be of a certain value or quality. If one party,
however innocently, leads another party to make a mistake as to the value of subject matter, it is
misrepresentation.
Law Of Contracts-I

Farrand vs Lazarus 2002 - A car dealer put a notice on a car that the mileage is incorrect even though he
knew that the reading was grossly incorrect. This was held to be misrepresentation.

Suppression of Material and Vital Facts


R vs Kylsant 1932- Company prospectus said that company was regularly paying dividends, which implied
that is was making profit. However, it did not say that company was making losses and dividends were being
paid from war time accumulated profits.

Expression of Opinion
Merely expressing an opinion is not misrepresentation.
Bisset vs Wilkinsen 1927 - The seller was aware that the land was being purchased for sheep farming and
he expressed an opinion that the land could carry 200 sheep. It turned out that the land was no suitable for
sheep farming. The seller was not held liable.

Section 19 says that any contract which is induced by Coercion, Fraud, or Misrepresentation is voidable at
the option of the party whose consent was caused due to coercion, fraud, or misrepresentation.
However, if the consent is obtained by misrepresentation of a fact or silence amounting to fraud, the contract
is not voidable if the party whose consent was so caused was able to discover it with due diligence.
Also, a fraud or misrepresentation that did not cause a party to give consent, does not render a contract
voidable.

Section 19 A says that when an agreement is created due to a consent induced by undue influence, such an
agreement is a contract voidable at the option of the contract whose consent is so caused.
Fraud - Section 17 Misrepresentation - Section 18
There is intention to deceive. No intention to deceive.
A contract involving fraud is liable for action in tort
No action in tort.
for damages.
Defence of ordinary diligence is not available for Can be defended on the ground that the person could find
fraud except for fraud by silence. out the truth by ordinary diligence.
Similarity in Fraud and Misrepresentation
1. Both contain false representation.
2. Both render a contract voidable.
3. In both the cases, the consent must have been caused due to that fraud or misrepresentation.
4. Defense of ordinary diligence is available to fraud by silence and misrepresentation.
5.

Q. 5 What are different types of mistakes? Explain their effects on the validity of the
contract.

There can be two types of mistakes - a mistake that misleads the parties to an agreement to consent and a
mistake that defeats the consent itself.

For example, A contracts with B for purchasing goods coming by a ship. However, unknown to both, the ship
has already drowned at the time of the contract. In this case, there was a mistake of a fact, which lead the
parties to consent. This type of mistake is covered by sections 20, 21, and 22.

In another example, A enters into a agreement with B thinking that he is C. In this case, there was no consent
from A at all because of mistake in identities. As another example, A agrees to sell to B his stock and B agrees
to pay 5000Rs for A's stock. However, A was thinking about his livestock (i.e. cattle) and B was thinking about
shares of a corporation. In this case, there was no consent because they agreed on the same thing but not in
in the same sense. These mistakes defeat the consent itself. There is no real consent here at all. This type of
mistakes is covered by section 13, which says that when two persons agree to the same thing in the same
sense, they are said to consent. Agreement on the same thing in the same sense is true consent and is called
consensus ad idem. If there is no consensus ad idem, there is no agreement, and hence no contract.

Section 20 says that an agreement is void when both the parties are under mistake as to matter of fact that is
essential to the contract.
Law Of Contracts-I

Illustrations
A agrees to buy a horse from B. At the time of agreement the horse was dead but no one knew about it. The
agreement is void.
A, being entitled an estate for the life of B, sells it to C. B was dead at the time of contract but both the parties
were ignorant of the fact. The agreement is void.

Section 21 says that a contract is not voidable if it was caused by mistake as to law in India. However,
mistake as to law outside India has the same effect as mistake of fact.

Section 22 says that contract is not voidable merely because one of the parties was under mistake as to fact.

Thus, for an agreement to be voidable due to a mistakes three conditions are required -

1. Both the parties are under mistake.


2. Mistake is of a fact.
3. fact is essential to the agreement.

Which facts are essential to the agreement?


This depends on the nature of the promise in each case. Something that is remotely linked to the agreement
is not an essential fact. For example, A agrees to hire B for taking his goods to a city. They came to know later
that there is no electricity. This fact is not essential to the contract. However, it they come to know later that all
transporters are on strike, this would be a fact essential to the agreement.

There are three types of things that are essential facts to an agreement - identity of the parties, identity and
nature of the subject matter of the contract, and nature and content of the promise itself.

Mistake as to identity
Mistake as to identity can occur due to
assumption of a false name
taking over a business
fraud

In the case of Cundy vs Lindsay 1878, a fraudster, who had a similar name as that of the defendant, sent an
order to the plaintiff . Plaintiff supplied the order which went to the defendant. It was held that since there was
a mistake as to identity, there was no contract between the parties.

In cases where identity of the offeree is important to the offerer, a contract cannot arise in the case of mistake
as to identity. In the case of Said vs Butt 1920, the plaintiff got the tickets for a show through a friend but the
defendant, the manager of the theater did not allow him to enter. It was held that since the manager did not
give the ticket for the plaintiff, there was no contract between them.

Mistake as to Subject matter


This can occur due to
Non existent subject matter
Mistake as to title or ownership
Different subject matters in mind
Mistake as to quality of subject matter.

Raffles vs Witchlehaus -Parties had different ships in mind but both were named peerless. It was held that
there was no consensus ad idem and so the contract must be set aside.
Smith vs Huges - Buyer wanted to buy old oats for his horse. The seller showed him the sample but didn't
say anything about the age. The buyer kept the sample for 24 hrs and then ordered. Later on he rejected the
order saying that the oats were new. It was held that buyer had no right to reject.

Mistake as to the nature of promise


Law Of Contracts-I

When a deed of one character is signed under the belief that it is of another character, the agreement is
wholly void.
Sarat Chandra vs Kanailal - A gift deed was signed under the impression that it is only power of attorney. It
was held void.

Limitations
Both Parties
According to Section 20 both the parties must be under a mistake for the agreement to be void. This is further
supplanted by article 22 that an agreement is not void if only one party is under mistake.

Erroneous Opinion
Explanation to section 20 says that an erroneous opinion regarding the subject matter does not render an
agreement void. This was reflected in the case of Smith vs Huges.

Mistake of fact and not of law


Section 2 1 says that mistake of law does not render an agreement void. Thus, the mistake must be of a fact.
A mistake of foreign law will be treated as a mistake of fact.

Q. 6 What is a lawful consideration? When would a consideration or object of an


agreement unlawful? Explain with illustration.

Q. What is a lawful consideration? When would a


consideration or object of an agreement unlawful?
Explain with illustration.
Section 23 says that a consideration or an object of an agreement is lawful unless,

1. it is prohibited by law.
2. it is of such nature that, if permitted, defeats the provisions of a law.
3. it is fraudulent.
4. it involves or implies injury to another person or property of another.
5. it is immoral or against public policy.

Illustrations
1. A promises to sell his house to B for 10000 Rs. The object is the house and the consideration is 10000/-
both are lawful.
2. A promises to pay B 1000/- if C fails to pay his debt to B within next 6 months. B upon this promise give 6
more months to C repaying debt.
3. A promises to B to superintend B's manufacture of Indigo, which is lawful, as well as a trade in illegal items
for a monthly salary of 5000/. Unlawful.
4. A promises to pay 5000/- per month to B to clean his house and live with him in an adulterous relationship.
Law Of Contracts-I

So on...

Forbidden by Law -
Means any law in force, including Hindu and Muslim personal laws.
Koteswar Vittal Kamath vs K Rangappa Baliga 1969 SC - Sale of liquor without license is void
and prices paid is irrecoverable.
Mannalal Khetan vs Kedar nath Khetan 1977 SC - If the intention of the law is to forbid something
in public interest, an agreement that contravenes it is void. However, if the intention is to merely
regulate something, the contract may not be void even if the parties have to pay a penalty.

Defeats the provisions of a law


1. Fateh Singh vs Sanval Singh 1878 - An accused was required to put a surety of 5000/- for good
behavior. He deposited the money with defendant and asked the defendant to become surety. Ofter
the period of surety, the accused sued to recover the deposit. Agreement was held void.
2. Regazzoni vs K C Sethia 1956 - Two parties made an agreement that one will supply jute to another
in an African country so that it can then be resold in another country to which export of jute bags was
prohibited. One party sued the other for breach of contract. Agreement was held void.

Fraudulent
1. Scott vs Brown Doering McNab and Co 1891 - A trader asked the broker to purchase a stock of a
company at a premium to create an impression in people that the company was worth paying a
premium. Later he discovered that the broker sold his own shares to him. The trader sued to revert
the transaction. Held void because it was done to defraud people.

Injury to person or property


1. Ram Sarup vs Bansi Mandar 1915 - An agreement said that a person would work for another
person for two years for borrowing rs 100. In case of default, he was to pay an exorbitant interest and
principal at once. This was held indistinguishable from bonded labor and this was injurious to person.
Held void.

Immoral
1. What is moral depends on the standards of morality prevailing at a particular time and approved by
the courts.
2. Interference in marital relations is immoral.
3. Dealings with sex workers
4. Allice Marry Hill vs William Clark 1905 - Adultery involving a married person is not only immoral but
illegal and any contract or promise related to that cannot be enforced.

Public Policy
1. Under Public Policy, sometimes the court may refuse to enforce a contract for the benefit of public
interest.
2. Ratanchand Hirachand vs Askar Navaz Jung 1976 - J Reddy of AP HC observed, "The twin
touchstones of public policy are advancement of public good and prevention of public mischief and
these are to be decided by the judges not as a men of legal learning but as experienced and
enlightened members of the society."
3. Trafficking in public offices, trading with enemy, interference with administration of justice, champerty,
marriage brokerage contracts, unfair or unreasonable dealings - when parties are not on equal
footing.
Law Of Contracts-I

Understanding of a lawful consideration is important because as per section24, an agreement is void if any
part of a single consideration for one or more objects, or if any one or any part of any one of several
considerations for a single object is unlawful.

Q. 7 "An Agreement without consideration is void." Explain this rule and state exceptions
if any.
Q. "An Agreement without consideration is void." Explain this rule and state
exceptions if any.
Indian Contract Act 1872 in section 2(e) says that every promise and every set of promises that form a
consideration for each other is an agreement. Thus, it is clear that the formation of consideration for a promise
or promises is a key ground on which a promise becomes an agreement. There cannot be an agreement if
there is no consideration. Section 25 of the act says the same thing in precise terms and also gives three
exceptions when an agreement without consideration is a valid contract:

Section 25: An agreement without consideration is void unless,


1. it is in writing and registered and the promise has been made due to natural love and affection
between the parties standing in near relation to each other.
2. it is a promise to compensate, wholely or in part, a person who has voluntarily done something for
the promisor or something that the promisor was legally bound to do.
3. it is a promise to pay for a time barred debt.

Natural Love and Affection

Rajlukhy Debi vs Bhootnath Mukherji - Court found no evidence of love.


Bhiwa vs Shivram - A person gave half of his property to his brother in order to be reconciled with him. Court
held that it was due to natural love and affection.
Past and Executed Consideration
The definition in 2(d) says "...has done or abstained from doing...". Thus, an act already done can be a valid
consideration. However, a past consideration and an executed consideration must be distinguished. For
example, if A saves B from drowning and if B promises to pay A 50/-, under English law, B is not bound by the
promise because there was no promise when the act was done. The act of saving is past consideration. On
the other hand, if A promises to pay 50/- to who ever finds his dog and if B finds and produces the dog, A is
bound to pay because the promise existed before the act. This is called executed consideration.

However, in Indian law, Section 25 (2) explicitly says that a promise to compensate a person who has
voluntarily done something for the promisor is binding. Thus, if B saves A from drowning and if A promises to
Law Of Contracts-I

pay B, then A is bound by the promise.

Further, in the case of a past service on request without any promise to pay, it is construed that there is an
implied promise to pay only the amount of payment is not fixed. Thus, a promise to pay for a past service
upon request is a valid contract.
In the case of Sri Sandhi Ganpatji vs Abraham, it was held that services rendered to a minor, which were
continued after his majority upon his request is a valid consideration for a promise to pay.

Value of the consideration


It is important that the consideration has some value in the eyes of law. If A promises to B to give his Rolls
Royce if B brings it from the garage, the promise is not binding because the consideration has no value in the
eyes of law. However, if A sells his horse worth 1000/- to B for 10/-, it is a valid consideration even if it is not
adequate provided that the consent was free. Explanation 2 of section 25 says that inadequate consideration
may be considered to be against free consent.
Haigh vs Brooks - A promise to pay for returning a document, which later on was found to be worthless, was
held to be a valid because the document was considered of some value at the time of the contract.

However, consideration need not be adequate.


De La Bere vs Pearson - A person lost money due to a financial advice given in a newspaper. The
newspaper was held liable because the consideration of buying the newspaper was of some value even if not
adequate.
Debi Radha Ranee vs Ram Dass - Forbearance to sue to sue is a valid consideration.

Performace of existing duties


In general performance of something that one was already required to do is not a valid consideration.

Performance of Legal Obligation


For example, a policeman is under legal obligation and performance of his duties cannot be a valid
consideration.

Performance of contractual Obligations


In the case of Ramchandra Chintaman vs Kalu Raju 1877, a lawyer was promised to get 100/- more if he
wins the case. The promise was held not binding because the lawyer was already under contractual duty to do
his best in the case.
However, a performance of a pre-existing contract with a third party was held a valid consideration. In the
case of Shadwell vs Shadwell, an uncle's promise to pay his nephew if he married some girl was held valid.
This was held by MP HC in the case of Gopal Co. vs Hazarilal Co AIR 1963.

Promise to pay less than the amount due.


Section 63 of Indian Contract Act says that payment of a smaller sum in satisfaction of a larger dept is valid if
this has been done under an agreement between the creditors and the debtors. It further gives an illustration
that if A owes B 5000 rs and if B accepts 2000Rs as a satisfaction of the whole amount at the time and place
where 5000 rs were due, the payment of 2000 rs discharges A of his debt.

Q. What are the important components of a consideration?


Section 2(d) defines consideration as follows:
When, at the desire of the promisor, the promisee or any other person, has done or has abstained from doing,
does or abstains from doing, or promises to to or abstain from doing, such an act or abstinence becomes a
consideration for the promise.
At the desire of the promisor
To be a valid consideration, the act must be at the desire of the promisor and not of anybody else. In the case
of Durga Prasad vs Baldeo 1880, the plaintiff had build the shops on the desire of the collector and not of the
defendants. Therefore the promise by defendants to pay a percentage of sales was held not binding.
In the case of Kedar Nath vs Gauri Mohd. 1886, the defendant had pledged 100/- for construction of town
hall. The plaintiff started work on that pledge and so the defendant was held liable to pay.

Provider of the consideration (Privity)


The British law has two principle governing the consideration.
1. The consideration must move from the promisee to the promisor.
Law Of Contracts-I

2. Only the person who is a party to the contract can sue for the performance.

Privity of Consideration
In India, the first rule is not followed at all. In fact section 2(d) specifically says that consideration can be
provided by the promisee or any other person. This was held in the case of Chinnaya vs Ramaya 1882.

Privity of Contract
In the case of Tweddle vs Atkinson 1882, it was held by the privy council that the person who is not a party
in the contract cannot sue. SC in the case of MC Chacko vs State Bank of Travancore 1969 has adopted
the same principle and held that the since the bank was not a party to the contract between the father and the
son, it cannot enforce the contract.
However, based on Privy Council's observation of the culture in terms of marriage and family relationship, in
the case of Kwaja Mohd. Khan vs Hussaini Begum 1910, some exceptions to this rule have been accepted.
1. Trust or Charge
When an agreement forms a trust for the benefit of a third person, the third person can enforce the
agreement. This was held in the case of Kwaja Mohd. Khan vs Hussaini Begum 1910 as well as in Rana
Uma Nath Bakhs Singh vs Jung Bahadur AIR 1938.
2. Marriage, partition, and other family matter
In the case of Daropti vs Jaspat Rai 1905, it was held that the wife was able to enforce the husband to fulfill
a promise that he gave to her father about providing her a separate residence.
3. Acknowledgement or Estoppel
Where by the terms of a contract a party is to make payments to a third party and the party acknowledges this
to the third party, a binding obligation is created towards him. This was held in the case of Devraja Urs vs
Ram Krishnaiya AIR 1952.

Q. 8 Agreements in restraint of Trade and agreement by way of wagering, are void. Explain
with leading cases.

Section 27 "Agreement is restraint of trade, void" says thus,


Every agreement by which any one is restrained from exercising lawful trade, business, or profession of any
kind is to that extent, void.

Exception: Saving of agreement not to carry on business, of which goodwill is sold.

Mahbub Chander vs Raj Koomar 1874 - Two shopkeeper entered into an agreement that one will pay the
other to close his business in that locality. One closed the shop but the other refused to pay. It was held that
the agreement was void. Since the wordings of section 27 do not do not use the word "absolute" as in section
28, even if the restraint is partial, it will be void.

Nordenfelt vs Maxim Nordenfelt Guns and Ammunition co ltd. 1894 - Inventor sold the goodwill of a gun
company to a buyer. The agreement was - Seller will not practice the same trade for 25 and the seller will not
do any business that will compete with the business carried on by the buyer at that time. It was held that the
first part is valid because it is reasonable but the second part is invalid because it is unreasonable.

English law tests reasonability while Indian law sees if it is allowed under statutory exceptions or exceptions
created by judicial decisions.

Collusion between Bidders and Tenderers


In general mutual arrangements between bidder so as to affect the final bid price are not considered in
restraint of trade.
Mohd Isack vs Daddepaneni AIR 1946 - Two persons entered into an agreement where by one would not
bid on a tender floated by Postal service and the other would pay him for that. The other person got the
contract but refused to pay. It was held that it was a valid contract.

However, now such matters should be considered with respect to Monopolies and Restrictive Trade Practices
Act 1969, which forbids such collusions.

Freedom of Press
An agreement that puts a restraint of press not to publish on the conduct of a person is void because it is
opposed to public policy.
Law Of Contracts-I

Restriction on Lease
Vidya Wati vs Hans Raj AIR 1993 - Lesor of a property can put a restriction on what kind of business can be
done on the property. It is an outlet of carrying business and not a restraint.

Exceptions
1. Sale of goodwill
2. Partnership : Under Partnership Act, partners of a firm may restrict their mutual liberty to do any
trade other than within their firm. An outgoing partner may also be restricted from carrying on similar trade for
a period of time.
3. Trade Combinations : Companies doing business in the same field may regulate their trade practices for
example opening and closing time of business even if they marginally put restraint. However, restrain on
employment are not allowed in disguise of regulation.
Korus Mfg vs Koluk Mfg 1959 - Companies made an agreement that they would not hire anybody who has
worked in the other company in past 5 yrs. Held void.
4. Exclusive dealing agreements
5. Restraint upon employees
Niranjan Shankar Golkari vs Century Spinning and Manufacturing Co 1976 - A company was offered
collaboration by a foreign company on the condition that they will maintain complete secrecy. A person was
employed in the company on the condition that he will not work for any other company in the same business
for 5 years. SC held the agreement valid.

Q. What is a wagering agreement? What are its


essential elements?
Section 30 - Agreement by way of Wager, void. -
Agreements by way of wager are void and no suit shall be brought for recovering anything alleged to be won,
or entrusted to anybody to abide by the result of an game or uncertain event on which a wager is made.

Exception - Any amount more than 500 rs can be paid to the winner or winners of any horse race.

Nothing in this section shall deem to legalize any transaction connected to horse racing to which provisions of
section 294-A of IPC apply.

Definition of wager was first given in Carlill vs Carbolic Smoke Ball Company 1892. It held that a wager
contract is one in which parties professing opposing views on the result of an uncertain event, mutually agree
that depending on the outcome of such event, one will pay or hand over a sum of money or other stake.
Neither party has any other interest in the event other than their stake that they may lose or win.

Essential elements -
1. Event must be uncertain : Outcome of an event cannot be predicted.
2. Each party must either win or lose depending on the result of an event

Baba Saheb vs Raja Ram 1940 : Two wrestlers agreed that if one fails to appear for a match he will pay Rs
500 to the other and the winner will take Rs 1125 out of the gate money. Defendant failed to appear and the
plaintiff sued for Rs 500. It was held that it was not wager because had the defendant appeared for the match
no one would have lost.
3. Parties should not have any control on the event
4. There should be no other consideration except the amount that one can win or lose - This is the
difference between a wagering agreement and an insurance agreement.

Speculative Transactions - An agreement to pay the difference between current market price and a
speculative price on a certain date is a wagering agreement. Such agreements are usually disguised as
Law Of Contracts-I

regular trade agreement and situation and facts of the case has to be looked into to decide whether it is a
wager or not. In the case of Kong Yee Lone vs Lowjee Namjee 1901, a trader promising to deliver 199000
bags of rice while he had no such capacity was held a wagering agreement.

Effects of wagering agreement - Such an agreement being void, it cannot be enforced by


the court.

Collateral Transactions - Wagering agreement is only void but not necessarily illegal, thus any agreement
such as a loan given to a person to pay a wagering debt can be enforced.
Gherulal Parek vs Mahadeodas Maiya AIR 1959 - A partnership to participate in wagering agreement is not
illegal and a parter who paid for wagering loses could sue other partners for contributing proportional funds.

Exceptions

1. Horse Race
2. Crosswords - Anything that requires skills to win. However, betting on a game being played by other people
is wagering.
Moore vs Elphic 1945 - Literary competitions which involve skill and effort is made to select the best and
most skillful competitor are not wagers.

Q. 9 What is meant by specific performance of a contract? What contracts can be


specifically enforced? What contracts cannot be specifically enforced? Can the following
be specifically enforced - contract to marry, a contract to give money on loan, an invitation
to dinner. What grounds may be taken by a defendant in a suit for specific performance of
the contract?

Q. What is meant by Specific Performance of a


Contract?
Specific performance is equitable relief granted by the courts in case of breach of contract in the form of a
judgement that the defendant must actually perform the contract according to its terms and stipulations. From
every contract arises an obligation for each party to do or to not do something. A breach of the contract by one
creates a moral right on the other to either enforce the performance of the contract or to get a satisfactory
compensation.
In many cases, a party to a contract is not interested in the compensation for breach of contract but the actual
object of the contract. In some case, no compensation can be considered enough. In such cases, law
provides a way to enforce the parties to actually fulfill their obligations. This is called "specific performance of
a contract".

The obligations may not necessarily arise from contract but may also arise from tort.

Q. What contracts can be specifically enforced?


Section 10 of Specific Relief Act 1963 specifies the conditions in which a contract can be specifically
enforced. These are as follows -

1. When there exists no standard for ascertaining the actual damage caused by non-performance of the
act agreed to be done.
2. When the act agreed to be done is such that a compensation in the form of money would not afford
adequate relief. Unless contrary is proved, the court shall presume that
1. the breach of a contract to transfer immovable property cannot be adequately relieved by
compensation in money.
2. the breach of a contract to transfer a movable property can be so relieved except
Law Of Contracts-I

1. when the property is not an ordinary article of commerce, or is of special value or


interest to the plaintiff or consists of goods which are not easily available in the
market.
2. where the property is held by the defendant as the agent or trustee of the plaintiff.

Nivarti Govind Ingale vs R B Patil 1997 SCC - A woman took a loan from a relative and executed a deed of
sale in favor of the relative's minor son with an agreement of re-conveyance at the repayment of loan. This
contract was held to be specifically enforceable. The relative had sold the property off to a buyer. This decree
was allowed to be enforced against such buyer also.

M S Madhusoodhanan vs Kerala Kaumudi Pvt. Ltd. 2003 SCC - Shares of a private company were held to
be goods of such a nature as are not easily obtainable in the market. Thus, SC allowed specific performance
to be granted in such cases.

Section 11 says that specific performance can be enforced when the act agreed to be done is wholly or
partly is in the performance of a trust. An exception is that the contract must not be in excess of the power of a
trustee.

Section 12 says that if, in the discretion of the court, only a small part of a contract cannot be specifically
performed and if such part can be alternatively compensated, the rest of the part can be specifically enforced.

According to Section 23, even if a contract includes a penalty or fixed amount of damages in case of default,
its specific performance can be ordered depending on the intention of the penalty. If the intention of the
compensation for damages is to secure the performance of the contract and not to give an alternative way of
fulfilling the contract, it can be specifically enforced.
This principle was adopted in the case of Manzoor Ahmed Magray vs. Ghulam Hasan Aram 1999 and M L
Devender Singh vs Syed Khaja 1973 by SC.

Q. What contracts cannot be specifically enforced?


Section 14 (1) of Specific Relief Act 1963 specifies the conditions in which a contract can be specifically
enforced. These are as follows -
a. when compensation in money is an adequate relief.
Ordinary contract to lend or borrow money, whether with or withour security, is an example of a
contract which cannot be specifically enforced.
Mennakshisundara vs Rathnasami 1918 - When a loan has already been advanced on the
understanding that a security will be provided against it, this can be specifically enforced.
b. when a contract runs into such minute and complex details or is dependent on personal qualifications
or volition of the defendant, or otherwise from its nature is such that a court cannot enforce specific
performance of its material terms.
Personal services such as painting, singing etc. cannot be specifically enforced. However, a contract
to publish a piece of music or to build a house can be specifically enforced because they are purely
mechanical functions.
c. when a contract is in its nature determinable i.e. can be brought to an end under given conditions.
Illustration - A and B enter into a partnership to do certain business, without specifying the duration of
the partnership. This cannot be specifically enforced because if enforced, either A or B might at once
dissolve the partnership.
A contract to employment is not specifically enforceable. The remedy in such cases is to sue for
damages.
Indian Oil Corp. vs Amritsar Gas Agency 1991 - A contract for distributorship cannot be specifically
enforced.
d. when a contract, the performance of which involves performance of continuous nature, which the
court cannot supervise.
Examples - An agreement to keep an airfield in operation, or an agreement by railway to keep signals
operating.
Rayner vs Stone 1792 - A tenant's undertaking to cultivate a farm in a specific way was held to be
not specifically enforceable.

Section 14(2) - A contract to refer a present or future dispute to an arbitration cannot be specifically enforced.
Section 14(3) - A contract to execute a mortgage or furnish any other security for repayment of a loan, which
Law Of Contracts-I

the borrower is not willing to repay at once.


Section 17 - A contract involving transfer of property when the party does not have the title or ownership of
the property.

Q. Can the following be specifically enforced - a contract to give


money on loan, contract to write a book, contract to marry, an
invitation to dinner, a contract to sell all goods of a class that a
party may require, a contract to run a franchised shop?
1. No, because as per 14 (1) (a), a contract that can be adequately compensated in money cannot be
specifically enforced.
2. No, because as per 14(1) (b), an act that depends on personal skills or volition of a party cannot be
specifically enforced. Here, it depends on personal skills.
3. No, because as per 14(1) (b), an act that depends on personal skills or volition of a party cannot be
specifically enforced. Here, it depends on personal volition.
4. No, because as per 14(1) (b), an act that depends on personal skills or volition of a party cannot be
specifically enforced. Here, it depends on personal volition.
4. No, because as per 14(1) (b), a contract that is too complex to be supervised by the court cannot be
specifically enforced.
5. No, because as per 14(1) (c), a contract that is determinable, i.e. can be ended, cannot be specifically
enforced. Here, a franchisee agreement can be terminated.

Q. What grounds may be taken by a defendant in a suit for


specific performance of the contract?
1. All the grounds upon which a contract is voidable - no free consent.
2. Plaintiff has not performed the whole or part of his part of contract.
3. All grounds in section 14 i.e. Compensation in money is adequate, Depends on personal
qualification, or determinable contract.
4. breach of trust or beyond its powers.
5. Contract when made gave unfair advantage to the plaintiff.
6. Involves hardship.
7. Plaintiff has chosen his remedy and obtained satisfaction for the alleged breach of contract.

s. Page
no Contract I - Part A No
1 Promise 1
2 Proposal 1
3 Valid offer 1
4 Invitation to offer & Standing offer 6
5 cross offer 8
6 Floating Offer 8
7 Acceptance 9
8 concensus ad idem 10
9 Standard form contract 19
10 Unilateral contract 20
11 Uberrimae fidei contracts 25
12 Past consideration 28
13 Adequacy ofconsideration 29
14 Privity of contract 31
Law Of Contracts-I

15 Stranger to a contract 31
16 Minor's agreement 35
17 Restitution 38
18 Capacity to contract 40
19 Lawful consent 46
20 coercion 47
21 Duress 47
22 undue influence 49
23 Mistake of fact 55
24 Legality of object 60
25 Immoral Agreement 60
26 Agreements in restraint of Marriage 65
27 Agreements in restraint of trade 67
28 Wagering contract 74
29 Contingent contract 75
30 void contract 77
31 Void and voidable agreement 77
32 discharge of contract 80
33 accord and satisfaction 80
34 impossibility of performance 82
35 Reciprocal promise 85
36 Performace of reciprocal promises 85
37 Frustration 90
38 Novation 97
39 Anticipatory breach 98
40 Quasi Contract 100
41 Finder of lost goods 100
42 Quantum meruit 106
43 Liquidated damages and penalty 108
44 Remotenesss of damage 111
45 Special Damages 113
46 Kinds of Damages 113
47 Nominal Damages 113
48 Declaratory decree 143
49 Injunction 145
50 Preventive relief
51 Pardahnashin women
52 Fundamental error

S.NO CONTRACT I - PART B Pg NO


1 Expalin the essential requisites of a valid contract 20
All agreements are not contracts but all contracts are
2
agreements - Discuss 21
Law Of Contracts-I

The law of contracts is not the whole law of agreement,nor is


3
it the whole law of obligations- comment 21
What is consideration how is it difined in the indian contract
4
act state the exceptions to the requirement of consideration 27
Define consideration and distinguish between consideration 27 TO
5
and privity of contracts 31
Stranger to a contract can not sue but a stranger to a
6 consideration can sue- explain . Bring out the differences
between the English law and Indian law 32-34
Who are competent to contract? What is the legal position of
7 the agreements made by an imcompetent person in Indian
and England 35-41
What is the consent and what are the factors vitiating free
8
consent 45-46
How is the validity of contract effected when free consent is
9
absent 45
What is public policy and give example of agreements
10
contrary to it 60
11 Discuss the law relating to contracts opposed to public policy 61
What do you understand by voidable contracts when a
12
contract becomes voidable 77
81 TO
13 Explain the different modes of discharge of contract
98
What is supervening impossibility unde what circumstances
14
the doctrine of supervening impossibility is not applicable 90
What is a quasi contract what types of quasi contracts are
15
recognized by the indian contract act 100
Explain the terms quasi contracts and state their
16 characteristics illustrate your answer by giving relevant
cases 100
Quasi contracts rests on the ground of equity that a person
17 shall not be allowed to enrich himself unjustly at the
expense of another- comment 100
18 Discuss the quasi contractual liability of a minor 100
Explain what contracts can be specifically enforced under 124 TO
19
the specific relief act 127
Explain with cases contracts which are not specifically
20
enforceable 130
21 The benefits of a contract cannot be assinged discuss
How far contracts entered into by the union government are
22
enforceable
23 Expalin the various modes of termination of proposal
Acceptance to an offer what a lighted match is to a train of
24
gunpowder- explain
Law Of Contracts-I

What is anticipatrory breach of contract? How does it differ


25
from actual breach ?
Explain the concept of frustration in the matters of contracts.
25 To what extent is the doctrine of frustration application to
mercantile transactions?

S.N Page Reference/cas


o CONTRACT PART -C NO e laws
The defendant and his wife were
enjoying leave in england.When the
defendant was due to return to
ceylon, where he was employed, his
wife was asked to remain in england Balfour Vs Balfour
by reason of her health.The defendant (Parties agreement
promised to send her an amount 30 in this case cannot
1 pounds a month towards the 2 be enforceable-
expenses of her maintenance.The however she can file
defendant sent the amount for some a case under family
time and later he stopped it due to law)
some misunderstanding between
them. Wife files a suit to recover the
arrears of maintenance promised by
the defendent.Advice the defendent.
General offer - Y is
X issued a pamphlet to pay entitled as he had
Rs.200000 to anyone who brings to the knowledge of the
2 5
him his missing son, Y traces the boy pamphlet -
and brings him to X. Can Y claim the Har Bhajanlal Vs
reward. Harcharanlal
Law Of Contracts-I

Naidu advertised in newspapers that


he would give Rs.100000 to any one
who brings back his missing
son.Ramana found that boy and Lalman Shukla Vs
handed over that boy to Naidu and Gauri Dutt ( Naidu
did not claim the reward as he had no need not pay as the
3 5
knowledge about the advt., at that servant had no
time.Later on Ramana came to know knowledge on the
about the reward and claimed for the newspaper offer)
reward from Naidu. Naidu refused to
pay.Should naidu pay? Explain the
principles.
X a bachelor agrees to marry a
spinster Y after death of his father Z .
Y cannot enforce X
4 But before Z's death X marries a 5
to marry
person other than Y. Y insists X for
marriage. Decide
kIran of delhi proposed to sell his
house ot Raju of Vishakhapatnam.
Raju sent his acceptance to post . Revocation of
Next day Kiran sends a telegram proposals of
withdrawing his acceptance. Examine acceptance may be
the validity of the acceptance in the done any time
5 16
light of following:- (a) The telegram of before the
revocation of acceptance was communication of its
received by kiran before the letter of acceptance but not
acceptance(b) The telegram of afterwards (sec 5)
revocation and letter of acceptance
both reached together
A telephoned to B to sell his plot for
Rs.25000 on a phone B arrived to A's
6
place with money A say that he has
droped the idea of selling . Advice B
A enter into a contract with B to
supply smuggled goods for Rs.20000.
B fails in supplying the same as he illegal contract and it
7
could not procure and did not return is void
Rs.20000 . A insists to repay
Rs.20000. Discuss
Privity of contract-
estoppel - A can
8 A makes an offer to C. B pretending to 22 & 56 claim from C
be C accepts the offer. Decide the Mistake of Identity-
validity of the offer Ingram Vs Little
9 A' voluntarily renders ervices to 'B'. Voluntary service is
'B' later unilaterally promises to pay not enforcable
'A' Rs 10,000 for the services
rendered by 'A'.'B' fails to keep up his
promise. 'A' sues 'B' for the recovery
of the promised amount. 'B' pleads
Law Of Contracts-I

lack of consideration for his


promise.Decide.
Mohirbibi Vs
Dharmodas Ghose
- no x is not libale as
the contract itself is
void ab initio and
10 21 & 36
X a minor represents to Y that he is a minor could not be
major and borrows Rs.1000 by estopped from
executing a promissory note. By the pleading minority
time the promissory note matures X Sadio Ali Khan Vs
becomes a major. Is X liable? Jai kishore
'A' sells to 'B' a minor. Certain goods
worth RS 55,000 on credit.The goods
Necessaries are
purchased by 'B' and their prices are
exception in minor's
a follows :
contract so cloths
11 (a) clothes Rs3000 (b) books Rs2000 37-38
and books cost are
(c) diamond rings Rs50000
recoverable and not
A wants to recover rs55ooo from B's
diamond ring
property.B pleads that the agreement
is absoltely void.Decide.
A applied to a banker for a loan at a
time where there is stringency in the This transaction in
money market.The banker declines to the ordinary course
make the loan except at an unusually of business and the
12 49
high rate of interest.A accepts the contract is not
loan on these terms.whetherthe induced by undue
contract is inducted by undue influence.
influence?Decide
'A' promise 'B' to drop a prosecution
illustration (h) - The
which he instituted against 'B' for
agreement is void ,
13 robbery and 'B' promises to restore 61
as its object is
the value of things taken. Is this
unlawful.
agreement enforceable?
X owes money to Y under a
contract.Later it is agreed that X has
Contingent contract-
to marry Z's sister of Y and X will not
14 76 illustration (c) . The
be liable. X agreed to do so but Z died
contract is void
in an accident.Discuss the liability of
X.
15 A' agreed to sell to 'B' 200 tons of 93 Frustration- A is not
potatoes to be grown on his farm at liable as crop
pune at a certain price.Although the disease or fire ,flood
potatoes were fully grown at the time etc natural calamity
of contract afterwards thy were beyond anyone's
destroyed due to disease and control.
therefore 'A' is unable to supply to 'B' (Satyabrata Ghose
Law Of Contracts-I

Vs Mugneeram
Bangur & co)
the potatoes he agreed upon.'A' seeks
your advice.Please advice him.
A has agreed to exihibit the film of B
in his cinema theatre. Due to
Specific ground of
unprecedented heavy rain two walls
Frustration -
of the cinema theatre collapsed,.
16 93 Destruction of the
License of the cinema theatre was
subject-matter
suspended , and hence A failed to
Taylor Vs Caldwell
exhibit the film to B. B files a suit
against A for breach of contract.
A a business man leaves goods at B B is bound to pay A.
house by mistake. The goods really Illustration (a) .
17 belong to C a neighbour of B . B treats 102 Obligation of person
the goods as his own. Disucsss the enjoying benefit of
liability of B non-gratuitious act
A' agrees to supply a machine to B on
a particular date, but fails to perform
his promise.B claims the following
ordinary damage
damages from A:
18 113 and special damage
(a).difference between the market
- A can cliam both
price and the agreement price.
(b)loss of the business profits.
Decide
Miss Anitha, a film actress agreed to
work exclusively for a period of two
years, for a film production production company
company.However, during the said can cliam damage
19 period she enters into a contract to 115 for the breach of
work for another film remaining period not
producer.Disscuss the rights of the worked Sec 75
production company under the Indian
Contract Act, 1872
The commissioner of a municial
corporation set out to construct a
town hall with the subscription of Kedarnath vs
people in the city.Mr. 'K' promised to Gouri Mohammad
20 contribute Rs.10,000 towards the RS 27 (Money spent based
expenses. As the construction is half on promise is
way through Mr. 'K' deneid his liability recoverable)
to pay the promised sum. Advise the
Municipal Corporation.
21 'A' promise to his 'B' to buy for him a
new motor cycle if 'B' secures first
class in his degree examination.'B'
works hard, spends sleepless nights
and secures first class. 'A' fails to
Law Of Contracts-I

keep up his promise. 'B' wants to


legally enforce the promise of 'A'.
Advicd 'A'.
X enters into a contract with Y to sell
a pistol without license. Y does so and
illegal contract and it
22 X is caught up by police. Discuss the
is void
liability of X and Y for entering into
such contract.
Fathers promised to pay his son a
sum of Rs.10 lakhs, if the son passes
LLB examiniation in first attempt. The
father and son offer
son passed the examination in the
is not enforcebale in
first attempt, but father fails to pay
23 contract ( Balfour vs
the amount as promised. Son files a
Balfour ) domestic
suit for recovery of the amount. State
offer
along with reasons whether son can
recover the amount under the indian
contract act 1872.
Raghu obtained Sunil 's consent to an
voiable contract -
agreement threatening Sunil to cause
Coercion , under
24 grievous hurt, later Sunil refused to
influence sunil is not
perform his part of the contract, Is
liable
Sunil liable? Advice
A orally agreed to sell his house for
Rs.100000 to B . Next day B
approached A to pay the said amount
25
to A refused to receive the amount
and told B that he had dropped the
idea of selling . Advice B
A film producer entered inot an Raj Rani Vs Prem
agreement with Shilpa, a girl aged 14 Adib - father can not
years, whereby Shilpa to act in a film . sue as it is being
26 Shilpa committed breach of 37 without
agreement due to which the consideration. Shilpa
producer loss. The film producer want can not sue as she is
to seek relief. Advice him minor - nullity
Rajendra a violinist entered into Impossibility after
contract to render a programme in formation of the
the theatre belonging to Kesav. contract-Rajendra
27 Afterwards Rajendra met with an RS 75 not liable- no
accident and was not able to perform remedy to kasav
the programme. Is there any remedy ( Rabinson Vs
to Kesav? Explain the principles. Devison)
A telegraphed to B to sell his house
for Rs.1500000. B reached A's house
B has not sent his
28 with cash but A refused to sell his
accpetance .
house.B insisting for the sale of
house. Advice proper course of action.
Law Of Contracts-I

X and Y are traders. X has a pvt


It is Valid . Sec 17
information of change prices which
illustration (d) - X is
29 will affect Y 's willingness to enter the 52
not bound to inform
contract.X keeps silent and enter into
Y
contract with Y.Is the contract valid?
A agrees to work as an assistant to B
for 3 years in B's clinic and not to
B can cliam damage
work elsewhere. But before 3 years A
for the breach of
30 wants to leave the job and take 115
remaining period not
another assignment. What are the B's
worked Sec 75
rights? Explain the principles invovled
in it?
Ramu a dealer in fruits delivers a
packet containing fruits at Raju's B is bound to pay A.
house by mistake instead of Illustration (a) .
31 delivering it at some other address 102 Obligation of person
who ordered for the fruits. Raju enjoying benefit of
consumes those fruits. Can Ramu non-gratuitious act
recover money?Decide
X has agreed to deliver the goods to Y
for certain price on 1-1-2012 Time is
ordinary damage
essence of the contract. X fails to
32 113 and special damage
deliver the goods on 1-12-2012. Y
- A can cliam both
rescinds the contract, and files a suit
to cliam damages. Decide
X and Y entered into an agreement
whereby X should pay Rs.100000 to Y
if it rains on specified day, otherwise, uncertain events -
33
Y should pay Rs. 100000 to X It rains contract void
on that specific day.Can Y recover
theamount from the court of law.
A contracts to pay a sum of money to
A is liable to pay to
B on a specific day. A does not pay
the extend of the
the amount on that day, as
34 115 principal sum and
consequence of non-payment . B is
interest up to the
totally ruined. Advice remedies
day of payment
availabel to B

The defendant and his wife were enjoying leave in england.When the
defendant was due to return to ceylon, where he was employed, his wife was
asked to remain in england by reason of her health.The defendant promised to
send her an amount 30 pounds a month towards the expenses of her
maintenance.The defendant sent the amount for some time and later he
stopped it due to some misunderstanding between them. Wife files a suit to
recover the arrears of maintenance promised by the defendent.Advice the
defendent.

Constituent of Proposal
Following are the requisites of a valid offer-
Law Of Contracts-I

1. Purposer or Offerer- proposer is the initiator of an agreement by way of proposal. According to


Section 2(a), proposer's has two purposes-
I. An expression of the offero's willingness to do or abstain from doing something.
II. Make a proposal to obtaining the assent of the offeree to the proposed act or abstinence.
Intention- Intention is not defined in Indian Contract Act 1872. But the offer initiated with the intention
of offeror. But Section 2(a) defines the similar word of intention which is Willingness. The proposal must be
made with willingness to do business on the proposed terms and conditions. Willingness shows that
intention to be bound by the proposal when accepted is an integral part of the concept of agreement. The
party who offers must know the legal consequences of the offer. An offer or its acceptance should be made
with the intention of creating legal relations. The test of intention is objective not subjective. The intentions
of the parties is naturally to be known the terms of the agreement and the surrounding circumstances.
Intention is the primary requisite of a valid offer, which later turns into a agreement, and
creates legal relations between the parties.
Balfour vs Balfour
Brief Fact-
W and H were husband and wife. H was employed in Shri Lanka. Both went to England for leave. There W
became ill and was compelled to stay in England. After completion leave H was to go to Shri Lanka. Before
going to Shri Lanka H promised to W to send 30 Ponds per month. Initial few months he do so but after that
he stop sending the money. W went to court for the probable expense of maintenance.
Judgment- The court rejected the claim.
Principle- it is not the contract when two parties agree to take a walk together or where there is an offer
and acceptance of hospitality. Second the parties was not intended that they shall be attended by
legal consequences
1. Legal consequences- an offer must be intended to create legal relations and must be capable of
creating legal relations. Offer is the first step of the agreement. When an agreement is enforceable by law it
becomes the contract. Offer is the starting point of a contract, it must be the capacity of legal binding.

1. Assent of other party- The proposal must be made with a view to obtaining assent of the other party.
Contract requires the consensus ad idem (consent at the same time).
2. Term- the terms of the contract must be certain and defined. There should be no ambiguity in the
terms and conditions of the offer.
3. Offer must be address- An offer may be addressed either to an individual or to a group of persons, or
to the world at large, and it may be made expressly or by conduct.
For example-
A offer to sell his house and gives an advertisement in the newspaper given the particular of the house, it is
the general offer.

A offer to sell his house to B for Rs 5000/- this is the specific offer.

Read more: http://www.articlesbase.com/law-articles/initialization-of-a-contract-


u-of-indian-contract-act-1872-3367670.html#ixzz1AbaP6xPQ
Under Creative Commons License: Attribution

The commissioner of a municial corporation set out to construct a town hall


with the subscription of people in the city.Mr. 'K' promised to contribute
rs.10,000 towards the expenses. As the construction is half way through Mr. 'k'
deneid his liability to pay the promised sum. Advise the Municipal Corporation.

The facts of this case were similar to those of the previous case, but the Secretary in this case incurred a
liability on the strength of the promise. Held, the amount could be recovered, as the promise resulted in
a sufficient detriment to the secretary. The promise could, however, be enforced only to the extent of
the liability incurred by the secretary. (Kedar Nath v/s Gouri Mohamed
Law Of Contracts-I

Pardanashin lady
Pardanashin lady means a lady who remains in parda due to tradition of family or society and generally she
do not come out of her home in general public and generally is illiterate.

If any pardanashin woman executes/makes any contract then she should be accompanied with any male
member of her family as her contract without this is not valid.
A contract with a woman who observes purdah or who is commonly referred to as pardanasheen is presumed
to have been influenced by undue influence. The law classifies the contracts made in the following situations
as invalid contract;

(1) A party to the contract being ignorant of the terms and the conditions of the contract.
(2) There being no independent counsel tp the party.
(3) A party to the contract not understanding the legal implications of her commitment under the contract.

Preventive relief
There can be cases where the nature of the contract do not allow damages to likely serve any purpose nor
admit to specific performance. In such cases, the court may have to restrain the party who threatens the
breach, to the possible extent. For example, a person undertakes a contract to sing at a particular place and
also undertakes not to sing anywhere else during the same period. In case the singer threatens breach, the
court cannot force him to sing. The positive side of the bargain is not specifically enforceable. But the
negative undertaking i.e. not to sing elsewhere, can be enforced by restraining him from singing elsewhere.
When he is prevented from resorting to other openings, it may exert some pressure upon his mind and he
may be persuaded to go ahead with the performance of his contract. This type of remedy is known as
preventive relief. This is granted by issuing an order known as injunction. Injunction is an order issued upon
the party concerned directing him/them to omit the performance of a particular duty or act. This is also
known as a mandatory injunction. Such relief is granted under the provisions of Part III of the Act. [3]

Important cases:-

Balfour Vs Balfour ( domestic contracts are not enforceable)

Taylor Vs Portington ( not enforceable if Terms are vague- it has to be certain)

Carlill Vs Carbolic Smoke Ball Co., ( Public offer enforceable)

Har Bajan Lal Vs Har Charan Lal ( Public offer enforceable)

Durga Prasad Vs Baldeo ( voluntary services done and cannot ask for consideration)
Law Of Contracts-I

Abdul Aziz Vs Masum Ali ( Donation promised but not given but it is not enforceable)

Kedarnath Vs Gorie Mohamamd ( expenses incurred against the donation promise- enforceable to the extend of expenditure)

Dunlop Pneumatic Tyre Co Ltd Vs Selfridege and co Ltd - Stranger to the contract cannot sue

Mohirbibi Vs Dharmadas Ghose ( Minor contract Void ab initio)

Sadio AliKhan Vs Jai Kishore ( minor cannot be estopped from pleading minority)

Kedhar nath Vs Ajundhia ( money given for marriage expenses was recoverable)s

Chikkam Ammiraju Vs Chikkam Seshamma ( made to accept by threatening suicide it is coercion)

Caravan Ellis Vs Cannon Ltd- Quatum Meruit ( Md is entitled to remuneration for the service rendered)

Potrebbero piacerti anche